Environmental Science Demo

Congratulations - you have completed .

You scored %%SCORE%% out of %%TOTAL%%.

Your performance has been rated as %%RATING%%


Your answers are highlighted below.
Question 1

A symbiotic relationship between two living organisms in which one organism benefits and the other is neither harmed nor helped is known as:

A
Commensalism
B
Predation
C
Parasitism
D
Mutualism
Question 1 Explanation: 
The correct answer is (A). In a mutualistic relationship, both organisms derive a benefit. In a commensalistic relationship, one organism benefits and the other is not substantially harmed or helped. In a parasitic relationship, one organism benefits at the expense of the other’s health. Predation occurs when one animal consumes another to extend its survival.
Question 2

Earthworms are a type of:

A
Scavenger
B
Parasite
C
Detritivore
D
Omnivore
Question 2 Explanation: 
The correct answer is (C). Detritivores obtain nutrients by consuming decomposing organic matter, or detritus. Scavengers obtain nutrients from carrion or animal waste; they do not possess the enzymes required to utilize detritus for nutrients. Parasites rely upon a living host for their nutrients. Omnivores consume both plants and animals.
Question 3

At which trophic level are cheetahs located?

A
Producers
B
Primary consumers
C
Secondary consumers
D
Detritivores
Question 3 Explanation: 
The correct answer is (C). Cheetahs are secondary consumers because they eat primary consumers, such as gazelle or hare.
Question 4

In which of the following ecosystems is net primary productivity the highest?

A
Temperate forest
B
Savanna
C
Swamps and marshes
D
Temperate grassland
Question 4 Explanation: 
The correct answer is (C). Net primary productivity is highest where the temperature is warm and where water and solar energy are most abundant.
Question 5

What percentage of biomass is conserved at each trophic level?

A
1%
B
5%
C
10%
D
20%
Question 5 Explanation: 
The correct answer is (C). This diagram shows that 10% of the energy is conserved from one trophic level to the next. For example, if the amount of grass in an ecosystem represents 10,000 joules, then in that same ecosystem, the energy contained in the primary consumer level will represent 1,000 J. This is because organisms use roughly about 90% of the energy they consume for cellular activities (such as respiration and growth).
Question 6

Which biogeochemical cycle depends on bacteria to make the nutrient usable by plants?

A
Nitrogen cycle
B
Carbon cycle
C
Phosphorus cycle
D
Water cycle
Question 6 Explanation: 
The correct answer is (A). The nitrogen cycle depends on nitrogen-fixing bacteria to convert atmospheric nitrogen into a form usable by plants.
Question 7

Which of the following terrestrial biomes is mostly made up of coniferous evergreen trees that can tolerate cold winters and short growing seasons, and is located in northern Europe, Russia, and North America?

A
Tundra
B
Boreal forest
C
Woodland/shrubland
D
Temperate seasonal forest
Question 7 Explanation: 
The correct answer is (B). Boreal forests are found between 50° and 60° N in Europe, Russia, and North America. Most vegetation is made up of coniferous trees.
Question 8

Which type of aquatic ecosystem is made up of salt-tolerant trees that are important in stabilizing tropical and subtropical coastlines?

A
Salt marshes
B
Intertidal zone
C
Mangrove swamps
D
Freshwater wetlands
Question 8 Explanation: 
The correct answer is (C). Mangroves are the dominant vegetation in mangrove swamps. They are salt-tolerant, help to stabilize tropical and subtropical coastlines, and provide an important habitat for marine organisms.
Question 9

At which level of complexity do different species interact with each other?

A
Ecosystem
B
Community
C
Population
D
Biome
Question 9 Explanation: 
The correct answer is (B). Communities are different populations of organisms living in the same location and interacting with one another.
Question 10

What percentage of solar energy striking producers is captured by photosynthesis?

A
1%
B
5%
C
95%
D
99%
Question 10 Explanation: 
The correct answer is (A). Typically, producers are only able to capture 1% of the solar energy available through photosynthesis.
Question 11

Which biogeochemical process is most impacted by burning fossil fuels?

A
Water cycle
B
Carbon cycle
C
Nitrogen cycle
D
Phosphorous cycle
Question 11 Explanation: 
The correct answer is (B). Burning fossil fuels releases carbon into the atmosphere. This increases atmospheric carbon concentrations and thus promotes global warming.
Question 12

Which of the following is released by volcanic eruption and is a necessary nutrient for living things but can also lead to acid rain?

A
Calcium
B
Potassium
C
Sulfur
D
Nitrogen
Question 12 Explanation: 
The correct answer is (C). Sulfur is an important nutrient. Volcanic eruptions are natural sources of sulfur dioxide, as are geothermal vents on the ocean floor. Burning fossil fuels also releases sulfur into the atmosphere, leading to acid precipitation.
Question 13

Which of the following biogeochemical processes has no atmospheric component?

A
Water cycle
B
Carbon cycle
C
Nitrogen cycle
D
Phosphorous cycle
Question 13 Explanation: 
The correct answer is (D). The phosphorus cycle has no atmospheric component. Phosphorous comes from the weathering of rocks and can find its way into surface waters, but it does not evaporate into the atmosphere in any way.
Question 14

Which biogeochemical cycle requires solar energy to move components from the earth into the atmosphere?

A
Water cycle
B
Phosphorus cycle
C
Nitrogen cycle
D
Carbon cycle
Question 14 Explanation: 
The correct answer is (A). The water cycle requires solar energy to evaporate water from the earth into the atmosphere.
Question 15

Which nutrient is a limiting nutrient in aquatic ecosystems and, when added to aquatic environments, typically causes algal blooms?

A
Hydrogen
B
Phosphorus
C
Potassium
D
Calcium
Question 15 Explanation: 
The correct answer is (B). Phosphorus is only added to the environment due to the weathering of rocks. Since this process is very slow, any amount of phosphorus added by humans can throw off the balance of an ecosystem and lead to eutrophication of surface waters.
Once you are finished, click the button below. Any items you have not completed will be marked incorrect. Get Results
There are 15 questions to complete.
List
Return
Shaded items are complete.
12345
678910
1112131415
End
Return

Congratulations - you have completed .

You scored %%SCORE%% out of %%TOTAL%%.

Your performance has been rated as %%RATING%%


Your answers are highlighted below.
Question 1

Which of the following tree species is a pioneer species in North American forests?

A
Beech
B
Aspen
C
Maple
D
Oak
Question 1 Explanation: 
The correct answer is (B). A pioneer species is a species of tree that can colonize new areas quickly and thrive. The aspen tree is often referred to as a pioneer species because of its ability to colonize new environments rapidly and maintain its longevity regardless of potential fires.
Question 2

Which of the following is the measurement used by ecologists to determine the biodiversity of a particular area?

A
Species richness
B
Species evenness
C
Genetic diversity
D
Ecosystem diversity
Question 2 Explanation: 
The correct answer is (A). The number of species in a given area, such as a grassland, is known as species richness.
Question 3

Which evolutionary process takes place when disturbance causes a dramatic decrease in the population size, causing the genetic composition of the survivors to substantially differ from the original group?

A
Mutation
B
Genetic drift
C
Founder effect
D
Bottleneck effect
Question 3 Explanation: 
The correct answer is (D). If a population is drastically reduced, the genetic composition may change. For example, if a natural disaster reduced the population to a small size, there will be fewer unique genotypes present in the remaining population. The offspring of these individuals will not be as genetically diverse due to the limited alleles present.
Question 4

At which level of complexity does evolution occur?

A
Ecosystem
B
Community
C
Population
D
Species
Question 4 Explanation: 
The correct answer is (C). A population is a group of individuals of the same species. Evolution requires genetic variation. Population is the lowest level of the organization of life at which such variation exists.
Question 5

Which of the following factors does not influence species richness of a community?

A
Habitat size
B
Distance from other communities
C
The length of time the habitat has existed
D
The amount of biomass
Question 5 Explanation: 
The correct answer is (D). Latitude influences the species richness—the further organisms are away from the North or South Pole, the species richness increases. The size of the habitat and the distance of that habitat from a source of colonizing species influence species richness as well. Species richness is also influenced by the length of time the habitat has existed, in that the longer the habitat has existed, the greater the species richness. However, amount of biomass in an ecosystem does not influence species richness.
Question 6

Organisms need to acquire energy to survive.  Which of the following are examples of adaptations that allow organisms to acquire the energy needed?

I. The long, slender beak of a hummingbird

II. The echolocation of a dolphin

III. The eyesight of an eagle

A
I only
B
II only
C
I and III
D
I, II, and III
Question 6 Explanation: 
All three choices are correct, so the correct answer is (D). The beak allows the hummingbird to reach nectar in deep flowers. The echolocation allows the dolphin to locate fish in murky water. The eyesight allows the eagle to spot food from far away.
Question 7

Which of the following would be a cultural service of an ecosystem?

A
Timber produced by trees
B
Food items like nuts and berries
C
Hiking paths to enjoy nature
D
Air purification by native plants
Question 7 Explanation: 
The correct answer is (C). Cultural ecosystem services are the benefits people gain from their interactions with different environmental spaces, such as woods or parks, and the activities, such as walking and cycling, they undertake in these spaces. The other three answer choices would be economic services provided by an ecosystem as they all have financial benefits for the people living near them.
Question 8
The finches from the Galapagos islands are descended from one ancestral species that arrived millions of years ago.  The primary difference between the species there now consists of different-sized and shaped beaks to use different food sources on the island.

Which of the following best describes why the ancestral finch species evolved into several distinct species after arriving on the islands?

A
Limited resources in the island ecosystems led to the adaptation of specialist traits to reduce competition between members of the species
B
An abundance of resources on the islands allowed the ancestral finches to be successful in filling all available niches
C
Reduced competition from other bird species allowed the finches to adapt to different food sources
D
Island climates tend to be more stable, which allowed the finches a consistent food source from year to year
Question 8 Explanation: 
The correct answer is (A). Islands tend to have fewer resources due to their smaller size and isolation from other sources of biodiversity. This lack of resources leads to adaptive radiation as a way to reduce competition and make use of the unoccupied niches.
Question 9

According to the theory of island biogeography, which two characteristics of islands should determine the levels of biodiversity found on them?

A
Size and soil type
B
Distance to the nearest source of biodiversity and the number of islands in the area
C
Number of islands in the area and soil type
D
Size and distance to the nearest source of biodiversity
Question 9 Explanation: 
The correct answer is (D). The size of an island determines the possible population size of species found there, the number of different ecosystems found there, and the types of species that can be found there. The distance to the nearest source of biodiversity determines how easily new species can make their way to the island. Less isolated islands see greater levels of immigration.
Question 10

Which of the following is a likely impact of the slash-and-burn style removal of the Amazon rainforest?

A
Increased CFCs reducing atmospheric ozone
B
Increased carbon sequestration
C
Decreased soil erosion compared to traditional logging
D
Increased average global temperatures due to higher atmospheric CO2
Question 10 Explanation: 
The correct answer is (D). Slash and burn puts the majority of the CO2 sequestered in the trees immediately into the atmosphere as well as removing the ability of the trees to continue sequestering more CO2 . Increased atmospheric CO2 leads to warmer global temperatures because it is a greenhouse gas that traps heat radiated from the surface of the Earth.
Question 11

Giant tube worms live on the ocean floor near hydrothermal vents. This environment experiences temperatures far hotter than most organisms can withstand and contains a high concentration of toxic chemicals such as hydrogen sulfide that escape from the vents. Which of the following statements best explains why the giant tube worm can survive in this environment?

A
Competitors of the giant tube worm were wiped out by pollution, allowing the worms to dominate the ecosystem
B
The toxic environment near the vents led to a trophic cascade that eliminated tube worm predators
C
Giant tube worms are well adapted to the environment near the vents and can thrive there outside the range of tolerance of other organisms
D
Giant tube worms can survive in any environment, so it makes sense that they would fill an available niche
Question 11 Explanation: 
The correct answer is (C). Giant tube worms have special adaptations that allow them to survive near-boiling water and use the bacteria that feed on hydrogen sulfide as food source.
Question 12

Based on the theory of island biogeography, why would a scientist expect to find less biodiversity on an island 25 kilometers from the mainland than on an island 8 kilometers from the mainland?

A
More species have the ability to travel to an island 8 kilometers away than to one 25 kilometers away
B
Islands closer to the mainland always have greater ecosystem diversity than islands further from the mainland
C
Islands further from the mainland normally have fewer available niches
D
Islands further from the mainland tend to see increased immigration as compared to islands closer to the mainland
Question 12 Explanation: 
The correct answer is (A). More organisms would have the ability to travel the shorter distance between the closer island and the mainland.
Question 13

A local ecologist has noticed that the population of bald eagles has fallen considerably over the past several years while other populations have remained stable. After running tests, the ecologist found that levels of DDT in fish in the area were extremely high. In this case, the bald eagles would be considered:

A
Pioneer species
B
Primary consumer
C
R-selected species
D
Indicator species
Question 13 Explanation: 
The correct answer is (D). The decrease in the bald eagle population served as an initial indication that something was wrong in the ecosystem. This led the ecologist to look for answers.
Question 14

Urban barn swallows have adapted to live under the bridges of busy expressways in major metropolitan areas. Below is a group of measurements of wing length from a population of barn swallows living under bridges and a population of barn swallows living in a rural setting.

Which of the following can explain the difference in wing length seen between the two populations?

A
Rural barn swallows need longer wings to better navigate the more natural ecosystem they are found in
B
Urban barn swallows need shorter wings to better avoid the fast-moving traffic under the bridges, whereas longer wings would make them less maneuverable and more likely to be struck by cars
C
Rural barn swallows have accumulated a different set of random mutations that led to longer wings
D
Urban barn swallows do not live long enough to grow to the larger-sized wings of the rural barn swallows
Question 14 Explanation: 
The correct answer is (B). The smaller wings of the urban swallows permit better maneuverability to evade traffic so that, over several generations, the swallows with shorter wings had a greater rate of survival. As a result, the population gradually had shorter and shorter average wing length.
Question 15

Which of the following is the best example of a keystone species in a pond ecosystem?

A
Bluegill
B
Stork
C
Dragonfly
D
Beaver
Question 15 Explanation: 
The correct answer is (D). Beavers help to create ponds by building dams in streams and creeks. The other species would then inhabit the ecosystem created by the beaver.
Once you are finished, click the button below. Any items you have not completed will be marked incorrect. Get Results
There are 15 questions to complete.
List
Return
Shaded items are complete.
12345
678910
1112131415
End
Return

Congratulations - you have completed .

You scored %%SCORE%% out of %%TOTAL%%.

Your performance has been rated as %%RATING%%


Your answers are highlighted below.
Question 1

Which of the following factors does NOT affect species richness?

A
Latitude
B
Survivorship curves
C
Time
D
Habitat size
Question 1 Explanation: 
The correct answer is (B). The species richness of a community is influenced by latitude, time, habitat size, and distance from other communities. Survivorship curves are NOT a factor that affects species richness. Survivorship curves are graphs that represent the proportion of surviving individuals as the cohort ages.
Question 2

In an experiment, two groups of paramecium species are grown together in a contained environment. P. aurelia grew well after 18 days, while P. caudatum declined to extinction over 18 days. This result demonstrates which of the following?

A
Competition
B
Predation
C
Competitive exclusion principle
D
Resource partitioning
Question 2 Explanation: 
The correct answer is (C). The competitive exclusion principle states that two species competing for the same limiting resource(s) cannot coexist. One of two species sharing identical, or nearly identical, niches will be driven to local extinction.
Question 3

A country has a net immigration rate of 3 per 1,000; a crude birth rate (CBR) of 9 per 1,000; and a crude death rate (CDR) of 11 per 1,000. What is the growth rate of this country?

A
0.1%
B
0.2%
C
0.5%
D
1%
Question 3 Explanation: 
The correct answer is (A). According to the question, CBR = 9 per 1,000; CDR = 11 per 1,000; and net immigration (which is equivalent to immigration minus emigration) at a rate of 3 per 1,000.

The national population growth rate is determined by calculating the difference between the factors contributing to population increase (CBR and immigration) and the factors contributing to population decrease (CDR and emigration), all divided by 10. Use the equation for national population growth:

Question 4

Which of the following describes the shape of the logistic growth model with time as the independent variable and population growth as the dependent variable?

A
Parabolic curve
B
J-shaped curve
C
S-shaped curve
D
Straight line
Question 4 Explanation: 
The correct answer is (C). The logistic growth model produces an S-shaped curve when graphed. This model illustrates a population that initially grows exponentially (curves sharply upward), slows (curves slowly to the right) as the population approaches the carrying capacity of the environment, and then levels off at carrying capacity.
Question 5

Which of the following is an r-adapted species?

A
Elephants
B
Mice
C
Gorillas
D
Wolves
Question 5 Explanation: 
The correct answer is (B). R-adapted species have adapted for high rates of growth. They are usually smaller in size, require little energy, produce many offspring, exhibit an early maturity and a short lifespan, and see most individuals die within a short period of time (though some individuals live for an extended period of time). K-adapted species have slower growth, are usually much larger, live longer lives, and have adapted to live at or near carrying capacity. Of the choices, mice most closely match the r-adapted species description.
Question 6

Which of the following population descriptions portrays the number of individuals in a particular age category?

A
Population size
B
Population density
C
Population distribution
D
Population age structure
Question 6 Explanation: 
The correct answer is (D). Within a population, there are individuals of varying ages. When population ecologists describe the number of individuals fitting into a particular age category, they are looking at population age structure. This data helps ecologists understand how quickly a population can grow.
Question 7

Questions 7-9 refer to the following graphs:

Which example fits the growth pattern shown in Graph II?

A
A population of 10 rabbits is introduced into an area with a good supply of food that regenerates easily
B
A population of 4 coyotes is reintroduced into an area that previously sustained a population of several hundred coyotes before they were hunted to local extinction
C
A population of reindeer was introduced to an island with no predators and a limited food supply; after the food was consumed, there was a dramatic drop in the population size
D
A population of harbor seals was actively hunted in Washington state in the early 20th century under a government program that viewed them as harmful predators, greatly reducing their numbers; since the program was discontinued, the seal population has rebounded in a logistic pattern
Question 7 Explanation: 
The correct answer is (C). Graph II shows a growing population with a sudden drop, indicating that the population grew exponentially and then saw a dramatic decline. This can be caused by the organism running out of food, causing mass starvation and death. Thus, the reindeer example is the best choice.
Question 8

Which letter on the graphs represents a population’s carrying capacity?

A
A
B
B
C
C
D
D
Question 8 Explanation: 
The correct answer is (B). When a population reaches carrying capacity, it stops growing and becomes stable.
Question 9

Which letter on the graph represents exponential growth?

A
A
B
B
C
C
D
D
Question 9 Explanation: 
The correct answer is (D). Exponential growth appears as a J-shaped curve on a graph.
Question 10

The average number of offspring required to offset the average number of deaths in a population so that the population remains stable is referred to as:

A
Crude birth rate
B
Crude death rate
C
Replacement level fertility
D
Total fertility rate
Question 10 Explanation: 
The correct answer is (C). Demographers calculate replacement level fertility to determine changes in population size. Replacement level fertility looks at the average number of offspring needed to offset the average number of deaths in a population. Typically, for a population of humans, this requires a bit over 2 offspring per woman.
Question 11

Question 11 refers to the following graph:

During which phase(s) of the demographic transition does the death rate decline while the birth rate remains high, causing the population to grow rapidly?

A
Phase 1
B
Phase 2
C
Phase 3
D
Phase 4
Question 11 Explanation: 
The correct answer is (B). In Phase 2, both the birth rate line and the death rate line are declining while the population size line is increasing.
Question 12

Question 12 refers to the following age structure diagram:

A country with an age structure like the one shown above will experience:

A
A rapidly growing population
B
Stable population growth
C
A decline in population
D
High life expectancy
Question 12 Explanation: 
The correct answer is (C). A country with an age structure diagram like the one above will experience a decline in the population because there are fewer children than adults. Therefore, when the children reach childbearing age, it is unlikely they will collectively produce enough offspring to offset their elderly, dying parents.
Question 13

Which of the following illustrates a global impact related to population growth?

A
Traffic into cities creates traffic jams and increases travel time into and out of developments
B
The conversion of forested areas into agricultural land reduces the uptake of carbon dioxide by plants
C
Converting land into agricultural land causes erosion
D
The building of new developments decreases habitat for local wildlife
Question 13 Explanation: 
The correct answer is (B). Conversion of forested areas into agricultural land reduces the uptake of carbon dioxide by plants, thereby not removing carbon dioxide from the atmosphere and contributing to global warming.
Question 14

Which line on the chart CORRECTLY matches the traits for K-selected and r-selected species?



A
A
B
B
C
C
D
D
Question 14 Explanation: 
The correct answer is (A). The following chart represents the correct traits of K-selected and r-selected species.

Question 15

Racoons are found in urban, suburban, rural, and wild settings. Because they can make use of a wide range of habitats and resources, they are considered:

A
Producers
B
Omnivores
C
Keystone species
D
Generalists
Question 15 Explanation: 
The correct answer is (D). Generalists can survive in a wide range of ecosystems and survive on a large variety of food sources.
Question 16

A factor that influences an individual's ability to survive and reproduce based on the size of a population in a given area is known as a:

A
Carrying capacity
B
Limiting resource
C
Density-independent factor
D
Density-dependent factor
Question 16 Explanation: 
The correct answer is (D). Density-dependent factors include things such as disease, where a larger population makes it easier for the disease to spread from one member to the next.
Question 17

A country with a high birth rate and a high death rate, where many children are born but few people make it to middle and old age, has an age structure diagram that looks like what?

A
A tapered column
B
A column
C
An inverted pyramid
D
A pyramid
Question 17 Explanation: 
The correct answer is (D). An age structure diagram displays the relative percent of a population at each age category. In a country with a high birth rate and a high death rate, the base of the pyramid, reflecting youth, would be widest, and there would be a steep rise to the top of the pyramid as more people die at a young age due to lack of healthcare and poor living conditions.
Question 18

According to the theory of demographic transition, a population in phase four would show which type of population growth?

A
Declining population growth
B
Stable population growth
C
Rapid population growth
D
Slow population growth
Question 18 Explanation: 
The correct answer is (A). A country in phase four of the demographic transition model would likely show a slowly declining rate of population growth and more affluent populations tend to have more women going to college and entering the workforce.
Question 19

Type I survivorship curves show that the number of survivors remains stable early but eventually decreases sharply at a particular upper age limit. Type II survivorship curves show that the number of survivors steadily decreases with an increase in age.

Which of the following can you determine from the descriptions of survivorship curves provided above?

A
Type II could represent an r-selected species with few surviving offspring
B
Type II could represent a K-selected species with few offspring that each have a high chance of survival
C
Type I could represent a prey animal with few offspring surviving to adulthood
D
Type II could represent a species that is in the middle of the r to K selection spectrum, such as a rabbit
Question 19 Explanation: 
The correct answer is (D). In a Type II survivorship curve, species typically have a middle-sized number of offspring that receive some parental care and thus have an increased chance of surviving to adulthood, but some factor, such as predation, leads to a steady decline in survivorship.
Question 20

Which of the following factors lead(s) to a declining birth rate in countries transitioning from phase two to phase three in the demographic transition model?

I. More women tend to stay home as a country becomes more developed, leading to decreased birth rates

II. More women tend to earn advanced degrees in developed nations, so they enter the workforce and have less time to raise children

III. There are more opportunities to own businesses in developed nations



A
I is correct
B
I and III are correct
C
I and II are correct
D
II and III are correct
Question 20 Explanation: 
The correct answer is (D). Developing nations tend to recognize the economic potential of women and their equality with men. As more women attain higher education and start their businesses, less time is devoted to having children.
Once you are finished, click the button below. Any items you have not completed will be marked incorrect. Get Results
There are 20 questions to complete.
List
Return
Shaded items are complete.
12345
678910
1112131415
1617181920
End
Return

Congratulations - you have completed .

You scored %%SCORE%% out of %%TOTAL%%.

Your performance has been rated as %%RATING%%


Your answers are highlighted below.
Question 1

An area beneath the ocean floor where tectonic plates move away from each other is known as a:

A
Subduction zone
B
Divergent plate boundary
C
Convergent plate boundary
D
Transform fault boundary
Question 1 Explanation: 
The correct answer is (B). Beneath the ocean floor, tectonic plates move away from each other at the divergent plate boundary. A subduction zone is where the tectonic plates pass over one another. Seafloor spreading does not describe the movement of the plates; instead, it describes the mantle pushing upward through to the surface. At a convergent plate boundary, the tectonic plates move toward one another, and at a transform fault boundary, the plates move past each other. The tectonic plates move away from each other due to the force of magma rising through and splitting the lithosphere, of which the tectonic plates are made.
Question 2

Which of the following layers of the earth is entirely liquid?

A
Inner core
B
Outer core
C
Mantle
D
Lithosphere
Question 2 Explanation: 
The correct answer is (B). The inner and outer core of the earth both consist of iron and nickel. Although the inner core is hot, its accompanying 330–360 gigapascal pressure keeps it in the solid phase. The outer core, although hot, lacks the pressure to keep it in the solid phase; consequently, it is the only layer of Earth in the liquid phase. The mantle is made up of magma, which is molten rock and not considered liquid. The asthenosphere is semi-molten but not liquid. The lithosphere is composed of both the earth’s crust and the outer mantle, both of which are solid.
Question 3

The loss of some or all of a soil’s ability to support plant growth is called

A
Erosion
B
Soil degradation
C
Physical weathering
D
Chemical weathering
Question 3 Explanation: 
The correct answer is (B). Soil degradation describes the loss of some or all of a soil’s ability to support plant growth resulting from biological, chemical, or physical deterioration. Weathering describes the breakdown of rocks and minerals, not soil. Erosion describes the process by which rock fragments and soils are removed from a landscape, but it does not specifically describe a soil’s capacity for supporting plant growth. Base saturation describes the proportion of soil bases to soil acids and does not specifically relate to soil-supported plant growth.
Question 4

Which list orders soil types from least to most organic content?

A
Young soil, immature soil, mature soil
B
Immature soil, young soil, mature soil
C
Mature soil, immature soil, young soil
D
Mature soil, young soil, immature soil
Question 4 Explanation: 
The correct answer is (B). As soil ages, the amount of organic material in it increases proportionally. Immature soil develops into young soil as plants and organisms die. Eventually, young soil becomes mature soil, which contains all of the organic material present in each of the earlier stages.
Question 5

Which list shows the layers of the atmosphere in order, starting with the layer closest to earth?

A
Stratosphere, troposphere, mesosphere, thermosphere
B
Troposphere, stratosphere, mesosphere, thermosphere
C
Stratosphere, troposphere, thermosphere, mesosphere
D
Troposphere, thermosphere, mesosphere, stratosphere
Question 5 Explanation: 
The correct answer is (B). The troposphere is the lowest portion of the atmosphere and ranges in depth from about 8 km – 14.5 km. The stratosphere is the second layer of the atmosphere and contains the ozone layer. The third layer, the mesosphere, is where most meteors burn up when entering the atmosphere. The thermosphere is the fourth layer and is where the International Space Station orbits.
Question 6

Which of the following correctly lists the layers of the earth from the innermost layer to the outside layer?

A
Solid inner core, liquid outer core, mantle, crust
B
Liquid outer core, solid inner core, mantle, crust
C
Solid inner core, liquid outer core, crust, mantle
D
Crust, mantle, liquid outer core, solid inner core
Question 6 Explanation: 
The correct answer is (A). The innermost layer of the earth is the solid inner core, followed by the liquid outer core, the mantle, and finally the crust.
Question 7

Which type of plate interaction occurs when plates move sideways past each other?

A
Fault zone
B
Divergent plate boundary
C
Convergent plate boundary
D
Transform plate boundary
Question 7 Explanation: 
The correct answer is (D). Transform plate boundaries occur when plates move sideways past each other.
Question 8

Which soil horizon is composed mainly of mineral material with very little organic matter?

A
O horizon
B
A horizon
C
B horizon
D
D horizon
Question 8 Explanation: 
The correct answer is (C). The B horizon consists mostly of minerals and very little organic matter.
Question 9

The percentages of the soil’s sand, silt, and clay is referred to as:

A
Texture
B
Minerals
C
Horizons
D
Relative particle size
Question 9 Explanation: 
The correct answer is (A). A soil’s texture is determined by the percentages of sand, silt, and clay contained in the soil.
Question 10

Where is the greatest amount of fresh water located on Earth?

A
Water bodies such as rivers, lakes, and ponds
B
The atmosphere
C
Oceans
D
Ice and glaciers
Question 10 Explanation: 
The correct answer is (D). No more than 3% of Earth’s water is fresh water. Fresh water sources are approximately distributed as follows:

0.04% atmospheric water
0.3% surface water
30% ground water
68+% ice and glaciers
Question 11

Which of the following atmospheric layers is the closest to the earth?

A
Thermosphere
B
Troposphere
C
Mesosphere
D
Stratosphere
Question 11 Explanation: 
The correct answer is (B). The following lists the atmospheric layer from the closest to the Earth increasing in altitude:

Troposphere
Stratosphere
Mesosphere
Thermosphere
Exosphere
Question 12

What could a farmer do to prevent crop loss due to heavy rains waterlogging his crops?

A
The farmer could amend his soil with clay to decrease porosity
B
The farmer could use contour plowing to prevent soil erosion
C
The farmer could use cover crops to increase organic material in the soil
D
The farmer could amend his soil with sand to increase porosity
Question 12 Explanation: 
The correct answer is (D). Amending the soil with sand would increase the porosity of the soil, allowing water to drain better and decreasing the chance of crops drowning. Clay would cause the water to sit on the surface even longer due to decreased porosity. Contour plowing would likewise help the water to sit in place longer. Increased organic material in soil also increases water-holding capacity.
Question 13

In the image above, which category would soil fall into if it is 30% sand, 10% clay, and 60% silt?

A
Silty clay
B
Loam
C
Silt Loam
D
Sandy Loam
Question 13 Explanation: 
The correct answer is (C). To read the above chart, start with the percent sand and find the intersecting line with clay and then silt.
Question 14

Which layer of the Earth’s atmosphere contains the highest percentage of water vapor?

A
The troposphere
B
The stratosphere
C
The mesosphere
D
The thermosphere
Question 14 Explanation: 
The correct answer is (A). Most weather occurs in the troposphere, where water evaporating from the surface of the Earth rises with warm air; as the air cools, the water condenses and falls back to the surface as precipitation.
Question 15

Which is the most abundant gas in the atmosphere by volume?

A
CO2
B
CH4
C
O2
D
N2
Question 15 Explanation: 
The correct answer is (D). 78% of our atmosphere is Nitrogen. Oxygen makes up 21%, and the rest of the gases account for around 1% by volume.
Question 16

Soybeans require soil that drains well to grow well. Which type of soil would be best to farm soybeans in?

A
Soil that contains at least 45% clay so that it retains plenty of water
B
Soil that contains at least 80% silt due to its low amounts of sand
C
Sandy loam that contains less than 20% clay
D
Silty clay loam because it contains all three soil components
Question 16 Explanation: 
The correct answer is (C). Clay has the lowest porosity of the three components of soil and prevents drainage. Sandy loam has higher amounts of sand and silt, allowing good drainage while still holding enough water for the soybeans.
Question 17

Which of the following has the greatest effect on the global distribution of solar energy that leads to seasons?

A
The tilt of the Earth’s axis concerning the sun
B
The curvature of the Earth
C
The shape of the Earth's orbit
D
The location of the Earth’s landmasses
Question 17 Explanation: 
The correct answer is (A). The tilt of the Earth’s axis is what drives the Earth’s seasons, as the hemisphere tilted towards the sun receives more solar energy than the hemisphere tilted away from the sun.
Question 18

Which agricultural practice would most likely lead to lower turbidity in nearby rivers and streams?

A
Using crop rotation in fields
B
Removing excess vegetation surrounding crop fields
C
Using synthetic fertilizers
D
Changing fields from crops to cattle pasture
Question 18 Explanation: 
The correct answer is (A). Crop rotation helps to reduce soil erosion. Increased soil erosion leads to more sediment in local waterways. Removing vegetation around the fields would also increase soil erosion, as would turning fields into cattle pasture. Use of synthetic fertilizers can lead to eutrophication in local waterways if excess N and P runoff from the fields makes its way into them.
Question 19

Though Iceland and Greenland are found at similar latitudes in the North Atlantic Ocean, Iceland sees a considerably warmer average daily temperature year round and is not covered by glaciers the way that Greenland is. Which of the following best explains the difference in the climates of Greenland and Iceland?

A
Solar radiation reaching Greenland is of higher intensity, leading it to more readily reflect off of the glaciers found there
B
Oceanic currents from equatorial regions flow around the coast of Iceland, helping to warm the atmosphere
C
The people living in Iceland have altered the climate to be more suitable
D
Polar air currents move at greater speeds over Greenland, stripping away much of the heat there
Question 19 Explanation: 
The correct answer is (B). Warm water currents flow up and around Iceland as part of thermohaline circulation. Cold arctic currents then flow down and around Greenland, having the opposite effect. Both countries receive a similar amount and intensity of solar radiation.
Question 20

Dams are built to regulate flooding downstream and occasionally to produce hydroelectric power. Which of the following is a negative environmental impact of building a dam on a river?

A
Sediment is prevented from flowing downstream, which can lower the productivity of portions of the river past the dam as well as any wetlands found at the mouth of the river
B
A new habitat will form behind the dam that can increase recreational spending in the area
C
More regulated flow downstream will stabilize the banks of the river
D
Local human populations will benefit from the increase in green, renewable energy
Question 20 Explanation: 
The correct answer is (A). Sediment carried downstream by rivers brings needed nutrients to the ecosystems found there.
Once you are finished, click the button below. Any items you have not completed will be marked incorrect. Get Results
There are 20 questions to complete.
List
Return
Shaded items are complete.
12345
678910
1112131415
1617181920
End
Return

Congratulations - you have completed .

You scored %%SCORE%% out of %%TOTAL%%.

Your performance has been rated as %%RATING%%


Your answers are highlighted below.
Question 1

If a deep well pumps water from an aquifer more rapidly than it can be regenerated, which of the following may form?

A
Cone of depression
B
Saltwater intrusion
C
Confined aquifer
D
Unconfined aquifer
Question 1 Explanation: 
The correct answer is (A). A cone of depression results when wells pump water at a rate faster than it can be renewed. When this happens, a depressed area, or “cone,” is formed due to the rapid withdrawal.
Question 2

Which of the following is the most efficient method of irrigation?

A
Drip
B
Flood
C
Spray
D
Furrow
Question 2 Explanation: 
The correct answer is (A). Drip irrigation is the most efficient form of irrigation because it utilizes a slowly dripping hose that is either laid on the ground or buried beneath the soil. It allows plants to absorb at a steady pace, rather than all at once, as in flood or furrow irrigation. The constant drip enables plants to maximize their water uptake, which leads to optimal growth.
Question 3

Deforestation can lead to:

A
Adverse soil erosion
B
Desertification
C
Extinction
D
All of the above
Question 3 Explanation: 
The correct answer is (D). Deforestation is the removal of a forest to convert the area to non-forest use such as farmland or urban use. The elimination of a forest has several negative consequences including soil erosion, desertification, extinction, and changes to climatic conditions. Soil erosion results from the absence of tree roots, which serve to maintain the soil’s integrity. Desertification and changes to climatic conditions both result from the ground’s increased exposure to the sun, and the lack of plant life contributing to the carbon cycle. As many animals rely upon the forest habitat to survive, the loss of the forest inevitably results in extinctions.
Question 4

Which of the following is a disadvantage of mono-cropping?

I. Soil depletion

II. Large expanses of land can be planted and then harvested

III. Crops are more vulnerable to attacks by pests

A
I only
B
II only
C
I and II only
D
I and III only
Question 4 Explanation: 
The correct answer is (D). Mono-cropping is an agricultural method that can lead to improved agricultural productivity by utilizing large plantings of a single species or variety. It allows for significant expanses of land to be planted and then harvested at the same time, increasing efficiency and productivity. However, it has several disadvantages including soil depletion and increased vulnerability to pests. This can result in a more fragile ecosystem with an increased dependence on pesticides and artificial fertilizers.
Question 5

What is bycatch?

A
A environmental problem caused by increased pesticide use.
B
Raising fish commercially in tanks or enclosures, usually for food.
C
Unwanted fish and other marine creatures caught during commercial fishing for a different species.
D
A common side effect of genetic engineering.
Question 5 Explanation: 
The correct answer is (C). Fish and other marine species that are incidentally captured often die in the process. Bycatch is one of the principal threats to marine biodiversity. There are two primary methods to reduce bycatch. One method is to ban fishing in areas where bycatch is unacceptably high. The other method is to use alternative fishing gear that can dramatically reduce the amount of bycatch.
Question 6

The phenomenon of urban blight has contributed to which of the following?

I. Degradation of the buildings and social environment of the city

II. Migration to the suburbs

III. Racial segregation

A
I only
B
II only
C
I and III only
D
I, II, and III
Question 6 Explanation: 
The correct answer is (D). “Urban blight” refers to the degradation of urban facilities and social environments. It leads to the exodus of populations from the urban centers to the suburban areas. All of the answer choices are consequences of urban blight. Urban blight has historically contributed to racial segregation, as much of the population leaving cities for the suburbs in the middle of the 20th century were predominantly middle to upper income Caucasians. This resulted in a larger minority population in the urban centers.
Question 7

Which of the following is a benefit of contour farming?

A
Allows vegetation of different heights to act as windbreaks and catch soil that might otherwise blow away
B
Prevents the same nutrients from being removed from the soil year after year
C
Conserves soil and prevents erosion
D
Reduces carbon dioxide emissions
Question 7 Explanation: 
The correct answer is (C). Contour farming is the practice of plowing and harvesting parallel to the contours of the land. It helps prevent soil erosion by slowing the flow of water across the field.
Question 8

Which of the following is a benefit of genetic engineering?

A
Genetic engineering decreases the likelihood of an allergic reaction to a specific type of food
B
The favorable genes in the genetically modified organism will spread to wild varieties and increase their ability to resist disease
C
Genetically modified organisms are healthier because their genes have been selected to make the crop more nutritious for humans
D
Genetically modified organisms need less pesticides because the plants have been developed to resist pests
Question 8 Explanation: 
The correct answer is (D). Genetically modified plants can be designed to resist pests. This means that GMOs may need less pesticides thus, ideally, reducing negative impacts to the environment.
Question 9

Abandoned coal mines can lead to which of the following environmental problems?

A
Increased photochemical smog from ozone seeping from the mine
B
Acid rain from increased SO2
C
Acid leaching from leftover mine tailings
D
Released nutrients leading to eutrophication of nearby waterways.
Question 9 Explanation: 
The correct answer is (C). Abandoned mines often contain large amounts of acidic waste rubble that can slowly leach into the surrounding environment as they fill with rainwater over time.
Question 10

Which of the following is a reason why biological control methods are not favorable to traditional pesticides?

A
Biological controls tend to target only one species where pesticides can kill beneficial organisms.
B
Biological controls do not add harmful chemicals that can spread into the local waterways because of run-off.
C
Crop loss to pests with biological controls is typically higher than crop loss with traditional pesticides.
D
Pests tend to adapt and become more resistant to traditional pesticides over time.
Question 10 Explanation: 
The correct answer is (C). Biological controls are often not as effective at controlling a pest population as the predator/prey relationship will reach a balance based on the carrying capacity of both species. If the introduced predator were to eliminate the pest the predator would then starve out as well opening the door for the reintroduction of the pest.
Question 11

Which of the following is NOT an environmental advantage of urbanization?

A
Less land is used per individual as compared to rural settings.
B
Lack of permeable surfaces causes a depletion of groundwater resources.
C
Individuals may be able to walk where they need to go or make use of public transportation to reduce fossil fuel reliance.
D
Less power is used per person as larger buildings are more efficient in retaining heat in the winter and cool in the summer.
Question 11 Explanation: 
The correct answer is (B). Urban areas tend to be dominated by blacktop and pavement that do not allow rainwater infiltration. This leads to reduced groundwater and increased runoff.
Question 12

It takes 20 times more land to produce beef than it does to produce corn. If corn can produce 1000 Kcal per hectare, how much land would it take to produce 5000 Kcal of beef?

A
20 hectares
B
100 hectares
C
200 hectares
D
2000 hectares
Question 12 Explanation: 
The correct answer is (B). five times the Kcal and twenty times the land. Five times twenty is 100 hectares.
Question 13

Which is likely why we have exceeded the maximum sustainable yield in ocean fisheries?

A
There are now too many marine fish farms
B
Interspecific competition is limiting population recovery in fish.
C
Too many reproductive-aged fish are being harvested yearly.
D
Climate change has negatively impacted fisheries.
Question 13 Explanation: 
The correct answer is (C). Industrial fishing practices decimate fisheries by efficiently catching large numbers of target species.
Question 14

Which piece of legislation is most effective at addressing the problems with overfishing?

A
The Resource Conservation and Recovery Act gives instructions on the management of aquatic resources.
B
The Delaney Clause of the Food, Drug, and Cosmetic Act regulates the harvesting of fish for human consumption.
C
The Clean Water Act regulates the dumping of pollutants into surface waters.
D
The Convention of International Trade in Endangered Species (CITES) of wild fauna and flora uses trade rules to protect commercially valuable species.
Question 14 Explanation: 
The correct answer is (D). CITES has far-reaching power over all nations to limit the trade of species that may be threatened by over-exploitation. The other laws are less inclusive or only govern actions in the U.S.
Question 15

What type of changes would be expected to be seen in a local hydrologic cycle as a result of clearcutting regional forests?

A
Siltation of local waterways will decrease
B
Transpiration from vegetation will increase
C
Runoff from watersheds will decrease
D
Evaporation from the soil will increase
Question 15 Explanation: 
The correct answer is (D). Clearcutting a forest decreases the albedo of the land which will lead to more heat and thus more evaporation. Tree roots also allow more water to infiltrate the soil reducing runoff. Lower amounts of runoff mean decreased siltation so clearcutting would increase both siltation and runoff.
Question 16

If a society wanted to be more sustainable they would:

A
Use energy more efficiently and reduce and reuse resources whenever possible.
B
Maintain current rates of energy consumption and resource use.
C
Convert the world's high energy resources into low-quality heat.
D
Use more nuclear power because it is a renewable resource.
Question 16 Explanation: 
The correct answer is (A). A sustainable society would reduce energy and resource use to a level that is replaceable by the planet. Current energy and resource use are not sustainable. Low-quality heat is wasted production. Nuclear energy is not renewable as Uranium is a limited resource.
Question 17

Which commercially used forestry method is most likely to cause fragmented landscapes and have negative impacts on biodiversity?

A
Slash and Burn
B
Shelter-wood cutting
C
Clear-cutting
D
Selective Cutting
Question 17 Explanation: 
The correct answer is (C). Clear-cutting removes all vegetation from an area. Slash and burn is almost as harmful but leaves behind nutrients in the soil from the burned vegetation. Selective cutting only removes old and sick trees. Shelter-wood cutting removes trees in stages giving time for saplings to begin growth where trees have already been removed.
Question 18
  Farm A Farm B
Animal Waste Manure reused as fertilizer Manure stored in lagoons
Average days in barn per year 0 days 45 days
Antibiotic use Only when sick Routinely to prevent spread of infection
     

Based on the table above, which best describes the two farms above?

A
Farm A is a free-range farm for cattle while farm B is a traditional CAFO for cattle.
B
Farm A is a free-range farm for chickens and farm B is a free-range farm for cattle.
C
Farm A is a free-range farm for chickens and farm B is an industrial poultry farm.
D
Farm A is a traditional CAFO for Cattle and farm B is a free-range farm for cattle.
Question 18 Explanation: 
The correct answer is (A). Concentrated Animal Feed Operations pack animals with antibiotics and require manure lagoons because of the denser population of animals. Poultry farms do not typically use manure lagoons.
Question 19

Of the following which, would be an unintended environmental impact of aquaculture?

A
Increased production of fish for market.
B
Concentrated organic waste in the surrounding water.
C
Decreased disease in the native fish population.
D
High cost of building the facility.
Question 19 Explanation: 
The correct answer is (B). Aquaculture crowds large numbers of fish into small pens. The dense population in the pen leads to increased local organic waste in the area near the aquaculture facility. This organic waste can lead to harmful algae blooms and possibly eutrophication.
Question 20

Which is a true statement about genetic diversity?

A
GM crops with decreased genetic diversity are better able to survive a changing environment.
B
Genetic diversity in a common crop such as soy beans is typically high.
C
Genetic resistance to disease can be increased by crossing crops with related varieties.
D
Genetic diversity is typically high in small populations.
Question 20 Explanation: 
The correct answer is (C). Decreased genetic diversity hinders a species' ability to adapt to a changing environment and to survive disease and predation. By crossing a crop with a related species you can increase genetic diversity and increase its chance of survival.
Once you are finished, click the button below. Any items you have not completed will be marked incorrect. Get Results
There are 20 questions to complete.
List
Return
Shaded items are complete.
12345
678910
1112131415
1617181920
End
Return

Congratulations - you have completed .

You scored %%SCORE%% out of %%TOTAL%%.

Your performance has been rated as %%RATING%%


Your answers are highlighted below.
Question 1

A stair-like structure that allows migrating fish to navigate through a dam is known as a(n)

A
Fish Ladder
B
Aqueduct
C
Dike
D
Levee
Question 1 Explanation: 
The correct answer is (A). Fish use a “fish ladder” or a “ladder” to make their way around a dam during migration. This structure functions like a set of stairs enabling fish to jump from landing to landing. The other answer choices refer to ways of transporting, damming, or storing water, not specifically to a method of assisting fish during migration.
Question 2

The largest renewable source of electricity generation in the United States is

A
Wind
B
Solar
C
Geothermal
D
Hydroelectric
Question 2 Explanation: 
The correct answer is (D). As of 2016, 15% of electricity generated in the US was from renewable sources. Hydroelectric accounted for 44% of renewable electricity generation. The most common type of hydroelectric power plant uses a dam on a river to store water in a reservoir. Water is released from the reservoir and flows through a turbine, spinning it, which in turn activates a generator to produce electricity. Here is the contribution of all sources:

Hydro 44%
Wind 37%
Biomass 10%
Solar 6%
Geothermal 3%
Question 3

Which of the following pairs is the predominant source of fuel for electricity generation in the United States?

A
Natural Gas and Oil
B
Oil and Coal
C
Nuclear fuels and Solar Power
D
Coal and Natural Gas
Question 3 Explanation: 
The correct answer is (D). The predominant sources of fuel for electricity generation in the United States are natural gas and coal. As of 2016, coal and natural gas combined to account for about 64% of the electricity produced in the United States. Nuclear accounts for approximately 20%.
Question 4

Which of the following is NOT one of the advantages of natural gas as an energy source?

A
Burning it releases no pollutants.
B
It is efficient for cooking and home heating.
C
It is abundant.
D
It can be stored and transported.
Question 4 Explanation: 
The correct answer is (A). The burning of natural gas does release pollutants. The advantage is that it burns cleaner than other fossil fuels, producing half the carbon dioxide as coal and about a third less than oil. It also emits fewer amounts of toxic chemicals like nitrous oxides and sulfur dioxide.
Question 5

Currently, most high-level radioactive waste from nuclear reactors in the United States is

A
stored in deep ocean trenches.
B
reprocessed into new fuel pellets.
C
chemically modified into safe materials.
D
stored at the power plant that produced it.
Question 5 Explanation: 
The correct answer is (D). Used nuclear fuel is stored at the nation's nuclear power plants in steel-lined, concrete pools filled with water or in massive, airtight steel or concrete-and-steel canisters. In the mid-1980s, plans were made to store the majority of the spent nuclear fuel at a central repository underneath Yucca Mountain in Nevada. But the project languished primarily due to opposition from Nevada residents who don’t want to import this dangerous material. Critics of the plan also worry that natural forces such as erosion and earthquakes could make the storage facility unstable. These nuclear isotopes can remain hazardous to humans for hundreds of thousands of years.
Question 6

Which of the following is true of passive solar designs?

A
These designs use mechanical and electrical devices for heating and cooling.
B
These designs have windows, walls, and floors that are made to collect, store, and distribute solar energy in the form of heat in the winter, and reject solar heat in the summer.
C
These designs have solar hot water systems which use pumps or fans to circulate fluid.
D
These designs use low-impact building materials and permeable concrete instead of conventional concrete to enhance the replenishment of ground water.
Question 6 Explanation: 
The correct answer is (B). Passive solar design uses sunlight without active mechanical systems. This is in contrast to active solar design which uses external sources of energy to power blowers, pumps and other types of equipment to collect, store and convert solar energy.
Question 7

Which of the following nonrenewable energy sources accounts for 68% of the CO2 that is emitted by the U.S. electric power sector?

A
Coal
B
Oil
C
Natural gas
D
Nuclear power
Question 7 Explanation: 
The correct answer is (A). Coal is used for about 30% of electric generation in the United States, making it the second most commonly used fuel. However, it accounts for 68% of carbon dioxide emissions. Natural gas is used to produce about 32% of US electricity but only accounts for 30% of CO2 emissions.
Question 8

Which type of coal has the lowest energy content?

A
Bituminous
B
Lignite
C
Anthracite
D
All of these have equal energy content
Question 8 Explanation: 
The correct answer is (B). Coals are ranked from low to high in this order: Lignite, sub-bituminous, bituminous, and anthracite. Lower-ranked coals are softer, lighter, contain less carbon, and have less energy content than higher-ranked coals.

Coal starts as peat. After a long amount of time, heat, and burial pressure, it is metamorphosed from peat to lignite. Lignite is called "immature" coal at this stage of development because it is rather light in color and remains soft. As time passes, lignite increases in maturity by becoming darker and harder and is then classified as sub-bituminous coal. With more time, pressure, and heat, more chemical and physical changes occur and the coal is classified as bituminous. At this point, the coal is dark and hard. Anthracite is the last of the classifications when the coal has matured completely. Anthracite coal is very hard and shiny.
Question 9

Which of the following nonrenewable energy sources is easily transported through establishing pipelines, produces a high net-energy yield, is subsidized by the U.S. government, and can be used to produce many other products such as paints, medicines and plastics?

A
Coal
B
Oil
C
Natural gas
D
Nuclear power
Question 9 Explanation: 
The correct answer is (B). Oil is also referred to as petroleum and crude oil in the United States. It is produced by the decomposition of organic material under high temperature and pressure for millions of years.
Question 10

Which of the following nonrenewable energy sources produces less SO2 and NOx than other fossil fuels when burned, leading to less of an impact on acid rain and photochemical smog?

A
Coal
B
Oil
C
Natural gas
D
Hydroelectric
Question 10 Explanation: 
The correct answer is (C). Natural gas is lighter than oil, and found above oil in petroleum deposits. Its two largest uses in the United States are electricity generation and industrial processes. Natural gas is considered a “clear fuel” because it contains fewer impurities and emits almost zero sulfur dioxide or particulates during consumption.
Question 11

In which part of a nuclear power plant is the fuel located?

A
Core
B
Cooling tower
C
Turbine
D
Electrical generator
Question 11 Explanation: 
The correct answer is (A). The core of a nuclear reactor contains up to 50,000 fuel rods.
Question 12

Which of the following is an advantage of nuclear power?

A
There is enough uranium left for the next 100,000 years
B
Nuclear wastes are easily recyclable
C
Uranium is a renewable resource
D
Nuclear power generation produces very little water pollution
Question 12 Explanation: 
The correct answer is (D). Nuclear power indeed produces very little water pollution. Estimates show that economically accessible uranium supplies could power nuclear reactors for about 200 years at current rates of consumption Nuclear power releases a small amount of CO2 (only 1/6 of the amount produced by fossil fuels). Nuclear wastes are not recyclable; they are very difficult to dispose of. Finally, uranium is not a renewable resource.
Question 13

Which of the following is an example of an active solar heating system?

A
Putting reflective coatings on roofs and exterior walls
B
Increasing insulation
C
Using photovoltaic solar cells to absorb solar energy
D
Installing skylights to allow sunlight into the building
Question 13 Explanation: 
The correct answer is (C). Active solar heating systems involve the use of solar collectors, such as photovoltaic solar cells, to absorb solar energy. The other methods listed are passive, in that they do not include any mechanical heating devices. Passive solar heating involves incorporating building designs that absorb heat and then slowly release it to maintain a consistent temperature in the building.
Question 14

Which of the following is the fastest growing source of electricity in the world?

A
Hydrogen fuel cells
B
Wind energy
C
Geothermal energy
D
Hydroelectricity
Question 14 Explanation: 
The correct answer is (B). Wind energy is the fastest-growing source of electricity in the world. It has risen from about 6.1 gigawatts in 1996 to nearly 540 gigawatts at the end of 2017.
Question 15

A family in Chicago wants to install solar panels on the roof of their house to help lower the electric bill.  Which direction should the solar panels face for the best possible results?

A
North
B
East
C
South
D
West
Question 15 Explanation: 
The correct answer is (C). Because Chicago is in the northern hemisphere the sun spends most of the time in the southern region of the sky. Having solar panels facing south would maximize the intensity of the sun's rays on the PV cells.
Question 16

In less developed countries, which fuel source is most commonly used for heating and cooking?

A
Coal
B
Oil
C
Natural Gas
D
Biomass
Question 16 Explanation: 
The correct answer is (D). Less developed countries rely on wood and dried animal waste for heating and cooking. Many developing countries do not have power grids and power plants. Families must burn material in their homes for heat and to cook food.
Question 17

Cars produce roughly 20 pounds of CO2 per gallon of gasoline burned.

If one person drives 30,000 miles in a car that averages 30 miles per gallon, while another drives 30,000 miles in a car that averages 20 miles per gallon, how many fewer pounds of CO2 were produced by the first car?

A
1000
B
10,000
C
60,000
D
600,000
Question 17 Explanation: 
The correct answer is (B). The first car would consume 1000 gallons of gas and the second car would consume 1500 gallons of gas. 20 X 1000 = 20,000 pounds of CO2 and 20 X 1500 = 30,000 pounds of CO2. 30,000 – 20,000 = 10,000 pounds of CO2.
Question 18

Which of the following is a negative environmental consequence of using photovoltaic solar cells?

A
Photovoltaic cells produce high amounts of NOx pollution while generating electricity.
B
Photovoltaic cells are responsible for stratospheric ozone depletion.
C
Photovoltaic cells are made using toxic metals that may get into the environment.
D
Photovoltaic cells contribute to the formation of tropospheric ozone and photochemical smog.
Question 18 Explanation: 
The correct answer is (C). Cadmium, Lead, and Aluminum are all used in the manufacture of PV cells. Once the PV cells need replacing it can be difficult to contain these metals without them entering the environment.
Question 19

A home uses five 100-watt lightbulbs for six hours per day.  How many kilowatt-hours of energy are used per year by using those lightbulbs?

A
1095
B
10,950
C
3,650
D
36,500
Question 19 Explanation: 
The correct answer is (A). five bulbs X 100 watts X six hours X 365 days / divided by 1000 watts per Kilo-watt.
Question 20

The extraction of natural gas is likely to have which negative impact on the environment?

A
Ozone Depletion
B
Photochemical Smog formation
C
Eutrophication
D
Groundwater contamination
Question 20 Explanation: 
The correct answer is (D). Extracting natural gas requires large amounts of water to be pumped into the ground under high pressure. The water is mixed with chemicals to help separate the natural gas found in sedimentary rock for easier extraction. Those chemicals can then leach into the surrounding groundwater.
Once you are finished, click the button below. Any items you have not completed will be marked incorrect. Get Results
There are 20 questions to complete.
List
Return
Shaded items are complete.
12345
678910
1112131415
1617181920
End
Return

Congratulations - you have completed .

You scored %%SCORE%% out of %%TOTAL%%.

Your performance has been rated as %%RATING%%


Your answers are highlighted below.
Question 1

Which of the following is an example of a secondary pollutant?

A
Ozone
B
Carbon Monoxide
C
Carbon Dioxide
D
Sulfur Dioxide
Question 1 Explanation: 
The correct answer is (A). Secondary pollutants are defined as pollutants that form as a result of atmospheric reactions involving a primary pollutant(s). Ozone is an example of a secondary pollutant because it results from a chemical reaction between two primary pollutants: nitrogen oxide and volatile organic compounds, in the presence of sunlight. The other answer choices are all examples of primary pollutants.
Question 2

The Montreal Protocol was signed in 1987 by 24 countries to regulate:

A
substances that cause air pollution.
B
substances that deplete the ozone layer.
C
indoor air quality.
D
the amount of smog in ppm in the air.
Question 2 Explanation: 
The correct answer is (B). The Montreal Protocol was a global effort designed to regulate substances that diminish the ozone layer. This has been a successful international agreement, and the ozone hole over Antarctica is slowly recovering. It is projected that the ozone layer will return to 1980 levels sometime between 2050 and 2070.
Question 3

The process of recycling a product into the same product is known as:

A
solid waste recycling
B
open-loop recycling
C
closed-loop recycling
D
reuse recycling
Question 3 Explanation: 
The correct answer is (C). Recycling processes are broadly classified into two categories: open-loop recycling and closed-loop recycling. With closed-loop recycling, end-of-life products are recycled into the same product. An example of closed-loop recycling is a used aluminum can that is recycled into a new aluminum can. Open-loop recycling is the process where material from one or more products is made into a new product, usually involving a change in the material itself. For instance, recycled plastic packaging might be used to make outdoor furniture.
Question 4

Which of the following is considered a point source of water pollution?

A
Sewage treatment plant discharging wastewater form a pipe into the ocean
B
Erosion from agricultural areas
C
Storm runoff from parking lots
D
Fertilizers from a golf course
Question 4 Explanation: 
The correct answer is (A). Point sources of pollution come from a distinct point of entry into a water body. Nonpoint sources come from more diffuse, large-spread areas of residential, agricultural, or urban areas.
Question 5

Which of the following sources of hazardous substances contaminate water when drinking water passes through pipes in older homes and can potentially damage the nervous system and kidneys?

A
Arsenic
B
Lead
C
Mercury
D
Acid deposition
Question 5 Explanation: 
The correct answer is (B). Lead is a heavy metal and a serious health threat. It is rarely found in natural water sources. Its main source is lead-lined pipes of older homes, brass fittings containing lead, and materials, such as solder, that are used to fasten pipes together.
Question 6

Which air pollutant bonds to hemoglobin, resulting in reduced air transport in the bloodstream?

A
Nitrogen oxides
B
Carbon monoxide
C
Carbon dioxide
D
Ozone
Question 6 Explanation: 
The correct answer is (B). Carbon monoxide is released from the incomplete the combustion of any kind. Carbon monoxide bonds to hemoglobin, interfering with oxygen transport in the bloodstream. It causes headaches in low concentrations and death in higher concentrations. Sulfur dioxide is a byproduct of combustion of fuels that contain sulfur, including coal, oil and gasoline. It is a respiratory irritant, harmful to plants, and contributes to acid precipitation. Carbon dioxide is a byproduct of burning fossil fuels, affecting climates and increasing greenhouse gasses. Ozone is a secondary pollutant formed by the combination of sunlight, water, oxygen, VOCs and NOx.
Question 7

What group of organisms is most affected by acid deposition?

A
Amphibians
B
Terrestrial mammals
C
Reptiles
D
Birds of prey
Question 7 Explanation: 
The correct answer is (A). Most amphibians cannot survive in water that has a pH less than 6.0. Amphibians are affected by acidity because they absorb water through their skin.
Question 8

Which of the following indoor air pollutants is a type of radioactive gas that seeps into a home through cracks in the foundation or soil, and may cause lung cancer?

A
Asbestos
B
Carbon monoxide
C
Radon
D
VOCs
Question 8 Explanation: 
The correct answer is (C). Radon is a radioactive gas that results from the decay of uranium. It seeps into homes through cracks in the foundation or soil. Human exposure to radon can cause lung cancer. Asbestos is a material used in insulation and causes respiratory diseases and lung cancer. Carbon monoxide, in addition to being an outdoor air pollutant, can also be an indoor air pollutant. Carbon monoxide binds with hemoglobin more efficiently than oxygen and reduces the amount of oxygen in the bloodstream. VOCs, or volatile organic compounds, are used in building materials, furniture, and other home products and are suspected to be carcinogenic. Mercury is a neurotoxin, which can damage the brain, kidneys, liver, and immune system.
Question 9

Which ingredient was removed from gasoline, and has significantly reduced the amount found in the atmosphere?

A
Carbon monoxide
B
Nitrogen oxide
C
Organic compounds
D
Lead
Question 9 Explanation: 
The correct answer is (D). Since lead was removed from gasoline, there has been a significant decline in lead found in the atmosphere.
Question 10

What is a concentration of 20 ppm equivalent to?

A
0.2%
B
0.02%
C
0.002%
D
0.0002%
Question 10 Explanation: 
The correct answer is (C). To change ppm (parts per million) to a percentage, simply move the decimal place four places to the left and add a % sign.
Question 11

Which is the strongest piece of evidence that a local area was experiencing acid deposition?

A
An increase in the amount of soluble heavy metals in a local pond.
B
A sudden die-off of all fish in a pond.
C
A gradual increase in the temperature of a local lake.
D
An increase in the diversity of micro-invertebrates in a local lake.
Question 11 Explanation: 
The correct answer is (A). Acid deposition causes toxic heavy metals to become more soluble by displacing naturally occurring ions in the soil.
Question 12

The average vehicle produces 1.39 grams of NOx per mile driven. If a truck is driven 32,000 miles per year for 12 years, how much NOx will that truck release?

A
534 grams
B
44,480 grams
C
444,800 grams
D
533,760 grams
Question 12 Explanation: 
The correct answer is (D).

1.39 grams × 32,000 miles × 12 years = 533,760 grams
Question 13

This substance is composed of fibers and is known to cause lung cancer:

A
Radon
B
Lead
C
Asbestos
D
Carbon Monoxide
Question 13 Explanation: 
The correct answer is (C). When asbestos is disturbed fragments may become airborne and enter a person's lungs if they are not wearing proper PPE. Radon can also cause lung cancer but is a gas while asbestos is a solid.
Question 14

This substance is emitted from manufactured building materials, furniture, and adhesives:

A
Radon
B
Formaldehyde
C
Asbestos
D
Lead
Question 14 Explanation: 
The correct answer is (B). Formaldehyde is a cancer-causing agent found in many building materials, especially glues used to hold furniture together and tiles to the floor. It is volatile and slowly disperses into the air of the places it is found.
Question 15

A pond in southern Canada had a pH of 8.2 in 1950, and by 1990 it’s pH had fallen to 5.6. Which of the following would be a short-term remediation strategy for the pond?

A
Change the pond ecosystem by introducing fish more tolerant of the lower pH.
B
Install barriers to prevent runoff from emptying into the pond.
C
Add calcium carbonate to the pond water.
D
Build taller smokestacks downwind from the pond.
Question 15 Explanation: 
The correct answer is (C). Calcium carbonate acts as a buffer that will return the pH to normal levels. This is not a long-term solution because it does not remedy the cause of the acid deposition.
Question 16

Which of the following is a natural source of CO2 in the atmosphere?

A
Photosynthesis by plants and plankton
B
Sedimentation of rock on the ocean floor
C
Decomposition of organic matter
D
Erosion of bedrock
Question 16 Explanation: 
The correct answer is (C). Aerobic decomposition uses the process of cellular respiration to break down organic matter. CO2 is a by-product of cellular respiration. CO2 is consumed in the process of photosynthesis and sequestered in the sedimentation of rock on the ocean floor.
Question 17

This toxin poses a risk to humans who eat large amounts of marine game fish:

A
Radon
B
Lead
C
Copper
D
Mercury
Question 17 Explanation: 
The correct answer is (D). Mercury builds up in the flesh of larger marine game fish through the process of biomagnification. Because mercury is fat-soluble it tends to stay in the bodies of animals that consume it. A larger fish that eats smaller fish over the course of it’s life ends up with the combined totals of all of the mercury from the smaller fish in them.
Question 18

The presence of which of the following in soil is most likely to mitigate the effects of acid rain?

A
Granite
B
Limestone
C
Sand
D
Humus
Question 18 Explanation: 
The correct answer is (B). Limestone is made of calcium carbonate which easily dissolves in acid and acts as a buffer raising the pH back towards neutral.
Once you are finished, click the button below. Any items you have not completed will be marked incorrect. Get Results
There are 18 questions to complete.
List
Return
Shaded items are complete.
12345
678910
1112131415
161718End
Return

Congratulations - you have completed .

You scored %%SCORE%% out of %%TOTAL%%.

Your performance has been rated as %%RATING%%


Your answers are highlighted below.
Question 1

Consumption of which hazardous material is known to affect the thyroid gland?

A
Lead
B
DDT
C
Mercury
D
Perchlorates
Question 1 Explanation: 
The correct answer is (D). Consumption of perchlorates is known to affect the thyroid gland by inhibiting the absorption of iodide and reducing thyroid hormone production. This lack of hormone inhibits the optimal function of the human body. The other hazardous materials mentioned here are deleterious to the human body, but are not known to specifically impede the function of the thyroid gland.
Question 2

The legislation that sets the national standards for safe drinking water is known as the:

A
Clean Water Act.
B
Safe Drinking Water Act.
C
Resource Conservation and Recovery Act.
D
Federal Water Pollution Control Act.
Question 2 Explanation: 
The correct answer is (B). The Safe Drinking Water Act sets the national standards for safe drinking water. The Clean Water Act is legislation that supports the “protection and propagation of fish, shellfish, and wildlife and recreation in and on the water” by maintaining and when necessary, restoring the chemical, physical, and biological properties of surface waters. The Resource Conservation and Recovery Act (RCRA) is the principal federal law in the United States governing the disposal of solid waste and hazardous waste. The Federal Water Pollution Control Act was the first major piece of legislation affecting water quality. The Water Quality Act protects and ensures the quality of surface and ground waters.
Question 3

What is the purpose of the leachate collection system in a modern sanitary landfill?

A
Extract methane produced for use as fuel.
B
Prevent additional water from entering the landfill.
C
Remove water and contaminants for treatment at a wastewater treatment plant.
D
Impede water flow from the landfill and retain positively charge ions such as metals.
Question 3 Explanation: 
The correct answer is (C). Recall that a leachate is a liquid that absorbs and carries particles of a material through which it passes. In a modern sanitary landfill, the leachate collection system’s function is to remove water and contaminants and transport them to a wastewater treatment plant.
Question 4

Which of the following is the most controversial method for oil spill cleanup?

A
Contain the spill using booms, and collect the oil from the surface of the water using skimmers
B
Use chemical dispersants to break down the oil
C
Add biological agents to the spill
D
Let the oil breakdown naturally
Question 4 Explanation: 
The correct answer is (B). Dispersants act by reducing the surface tension that stops oil and water from mixing. Smaller droplets of oil are then formed, which helps promote rapid dilution of the oil by water movements. In theory, this allows the oil to be more rapidly degraded by bacteria (bioremediation) and prevents it from accumulating on beaches and in marshes. However, studies have indicated that chemical dispersants are toxic to marine life. A dispersant called Corexit was used in unprecedented quantities during the 2010 Deepwater Horizon oil spill in the Gulf of Mexico. This was highly controversial and the long-term environmental effects of this are still being examined.
Question 5

Superfund is the common name given to which federal law?

A
The Clean Air Act
B
The Clean Water Act
C
The Comprehensive Environmental Response, Compensation, and Liability Act
D
The Resource Conservation and Recovery Act
Question 5 Explanation: 
The correct answer is (C). Passed in 1980, Superfund was designed to clean up sites that were contaminated with hazardous waste. It is also known as CERCLA.
Question 6

Electronic waste such as cathode ray tube televisions and computer monitors may contain this toxic metal.

A
Cadmium
B
Aluminum
C
Copper
D
Titanium
Question 6 Explanation: 
The correct answer is (A). Cathode ray tube (CRT) televisions and computer monitors may contain toxic metals such as lead, mercury, and cadmium.
Question 7

Which Act is the principal federal law governing the disposal of solid waste and hazardous waste?

A
Resource Conservation and Recovery Act
B
Superfund Amendments and Reauthorization Act
C
Toxic Substances Control Act
D
Occupational Safety and Health Act
Question 7 Explanation: 
The correct answer is (A). Enacted in 1976, the RCRA set federal standards for the treatment, storage and disposal of hazardous waste in the United States.
Question 8

The increasing concentration of a toxic substance in the tissues of organisms at successively higher levels of the food chain is known as:

A
Biomagnification
B
Bioaccumulation
C
Biodilution
D
Bioconcentration
Question 8 Explanation: 
The correct answer is (A). Biomagnification occurs across food chain levels when a producer or primary consumer ingests a toxic substance and is in turn consumed by a predator. At each level of the food chain, the concentration of the toxic substance increases. Bioaccumulation and Bioconcentration occur within an organism. Biodilution is the opposite of biomagnification.
Question 9

Which type of carcinogenic synthetic organic compound were manufactured in plastics and insulating electrical transformers until 1979, and even though they are no longer manufactured in the United States, they are still present in the environment?

A
Dichlorodiphenyltrichloroethane
B
Perchlorates
C
Polychlorinated biphenyls
D
Polybrominated diphenyl ethers
Question 9 Explanation: 
The correct answer is (C). Polychlorinated biphenyls, or PCBs, are a group of industrial compounds that have caused many environmental problems. Until 1979 they were manufactured in plastics and insulating electrical transformers. Even though they are no longer manufactured, they are still present in the environment because of their long-term persistence.

Dichlorodiphenyltrichloroethane, or DDT, is an insecticide designed to target nerve transmissions in pests. It was found that DDT moved up the aquatic food chain up to eagles, causing eagles to produce thin shells that broke before the embryo developed.

Perchlorates are used for rocket fuel and are easily leached from contaminated soil into the groundwater where they can persist for many years. Perchlorates can affect the thyroid gland and reduce important hormone production.

Polybrominated diphenyl (PBDEs) ethers are flame retardants added to a variety of items including construction materials, furniture, and clothing. Since the 1990s, scientists have been detecting them in fish, aquatic birds, and human breast milk. Exposure to some types of PBDEs can lead to brain damage. Several states, including Washington and California, have banned the manufacture of several types of PBDEs. Pharmaceuticals are not components of plastics and insulating electrical transformers.
Question 10

Which of the following contributes to thermal pollution?

A
Geothermal energy production
B
Hydroelectric energy production
C
Cultural eutrophication
D
Electric power plants
Question 10 Explanation: 
The correct answer is (D). Electric power plants use water for cooling to cool the steam converted from water back to water again. This is done by bringing in cold water from river, lakes or oceans to cool the steam. The water is then returned back to nature at a significantly warmer temperature. Hydroelectric energy production does not create thermal pollution. Geothermal energy production uses the energy from the earth to heat homes. Cultural eutrophication is the process of nutrients polluting surface water and the cascade of events that follow. Heat islands are urban areas that are up to 10°F warmer that surrounding areas.
Question 11

During the process of sewage treatment, which stage removes large objects, such as leaves, diapers, tampons, and wet-wipes?

A
Pretreatment
B
Primary Treatment
C
Secondary Treatment
D
Tertiary Treatment
Question 11 Explanation: 
The correct answer is (A). The pretreatment stage removes large debris as well as grit. This is followed by primary treatment, which uses settlement tanks to separate organic solid matter. Secondary treatment then removes dissolved and suspended biological matter with the help of water-borne microorganisms. Tertiary treatment is the final treatment and typically includes a disinfection process. Autolysis is not a stage of sewage treatment.
Question 12

Which group of harmful chemicals interfere with the normal functioning of hormones in an animal’s body?

A
Neurotoxins
B
Carcinogens
C
Endocrine Disruptors
D
Allergens
Question 12 Explanation: 
The correct answer is (C). Endocrine disruptors are chemicals that interfere with the normal functioning of hormones in an animal’s body. Neurotoxins are chemicals that disrupt the nervous systems of animals. Carcinogens are chemicals that cause cancer. Teratogens are chemicals that interfere with the normal development of embryos or fetuses. Allergens are chemicals that cause allergic reactions.
Question 13

Which of the following is an example of solid waste pollution?

A
Sediment
B
Oil
C
Mercury
D
Garbage
Question 13 Explanation: 
The correct answer is (D). Solid waste includes materials discarded from homes and industries that do not pose a toxic hazard to organisms.
Question 14

All of the following might be used to reduce the effects of eutrophication in a lake EXCEPT:

A
Adding Nitrates
B
Dredging the lake to make it deeper.
C
Introducing insects to consume nuisance plants
D
Pumping oxygen into the lowest layers of the water
Question 14 Explanation: 
The correct answer is (A). Adding nitrates would accelerate the process of eutrophication. Dredging the lake removes built-up waste that causes eutrophication. Insects that consume nuisance algae allow sunlight to reach deeper into the lake. Pumping oxygen into the deeper layers of the lake reverses the impact of eutrophication.
Question 15

Which is an example of a non-point source of pollution?

A
Dumping from a processing plant
B
Exhaust pipe from a generator
C
Outflow from a waste treatment plant
D
Agricultural runoff
Question 15 Explanation: 
The correct answer is (D). Agricultural runoff gathers from all regional agricultural fields and builds up in local waterways. The other examples are all easily identifiable sources you can “point” to.
Question 16

Testing the tissues of an average US citizen would likely show the presence of DDT. DDT was banned in 1972. Why would DDT still be found in people today?

A
Newer pesticides break down into DDT and then enter the ecosystem
B
Other countries still use DDT on their crops
C
DDT is water-soluble
D
DDT is naturally occurring in many bodies of water
Question 16 Explanation: 
The correct answer is (B). The US imports crops from many other countries that still use DDT as a pesticide. DDT is not water-soluble, is not naturally occurring, and is not the result of newer pesticides breaking down.
Question 17

Bioaccumulation occurs because:

A
The substance is fat-soluble and will build up in tissues faster than it is removed.
B
Organisms tend to consume more of the substance than other things.
C
The substance is water-soluble and quickly removed from an organism.
D
The substance is beneficial to organisms lower on the food chain.
Question 17 Explanation: 
The correct answer is (A). Fat-soluble substances tend to gradually build up in a bodies fat reserves. Because they are not water-soluble, these substances are not typically removed by the kidneys or the liver.
Question 18

LD50 refers to:

A
The dose at which a toxin harms 50% of the individuals in a population.
B
The length of time needed for a drug work on individuals in a population.
C
The dose at which 50% of exposed individuals die in a population.
D
The length of time needed for 50% of exposed individuals to die in a population.
Question 18 Explanation: 
The correct answer is (C). LD50 stands for Lethal Dose 50%. LD50 and ED50 (effective dose 50%) are used to predict safe concentrations for humans. Typically the LD50 is divided by 1000 to get the value that may be safe for human exposure.
Once you are finished, click the button below. Any items you have not completed will be marked incorrect. Get Results
There are 18 questions to complete.
List
Return
Shaded items are complete.
12345
678910
1112131415
161718End
Return

Congratulations - you have completed .

You scored %%SCORE%% out of %%TOTAL%%.

Your performance has been rated as %%RATING%%


Your answers are highlighted below.
Question 1

Which of the following greenhouse gases is the longest-lasting in the atmosphere?

A
Carbon dioxide
B
Methane
C
Nitrous oxide
D
Chlorofluorocarbons
Question 1 Explanation: 
The correct answer is (D). Different chlorofluorocarbons (CFCs) can remain in the atmosphere for different lengths of time, some for around 140 years. They have a global warming potential ranging from less than 5,000 to over 13,500.
Question 2

Which of the following is NOT a greenhouse gas?

A
Carbon dioxide
B
Water vapor
C
Nitrogen
D
Methane
Question 2 Explanation: 
The correct answer is (C). A greenhouse gas is a gas in the atmosphere that absorbs and emits radiation within the thermal infrared range. This process causes the greenhouse effect. The primary greenhouse gases in our atmosphere are water vapor, carbon dioxide, methane, nitrous oxide, and ozone. Nitrogen, oxygen, and argon are not greenhouse gases.
Question 3

Which of the following has the greatest Global Warming Potential (GWP)?

A
Carbon dioxide
B
Methane
C
Nitrous oxide
D
Fluorinated gases
Question 3 Explanation: 
The correct answer is (D). Fluorinated gases (such as HFCs and PFCs) are a group of man-made gases used in a range of industrial applications. Because they do not damage the ozone layer, they are often used as substitutes for ozone-depleting substances. However, fluorinated gases are powerful greenhouse gases, with a global warming effect that can be thousands of time times greater than that of carbon dioxide.
Question 4

Which of the following is a potential result of higher average air temperatures?

A
Increase in the frequency or severity of storms
B
Increases in the surface water and/or groundwater inputs
C
Increases in flooding and associated water runoff
D
All of the above
Question 4 Explanation: 
The correct answer is (D). Increased average air temperatures can cause these, and many other effects, on the Earth.
Question 5

The melting of glaciers causes which of the following?

A
Landslides
B
Flash floods
C
Displacement of people who depend on glacial melting as a water source
D
All of the above
Question 5 Explanation: 
The correct answer is (D). In addition to the list above, glacial melting also increases variation in water flows into rivers.
Question 6

The International Union for Conservation of Nature uses five categories to define the status of a species. Which category refers to a group of species that is widespread and abundant?

A
Extinct
B
Threatened
C
Near-threatened
D
Least concern
Question 6 Explanation: 
The correct answer is (D). The least concern status is for species that are widespread and abundant. Data-deficient species have no reliable data to assess their status. Extinct species are those that were known to exist as recently as the year 1500 but no longer exist. Threatened species have a high risk of extinction in the future. Near-threatened species are very likely to become threatened in the near future.
Question 7

In the 1600s, honeybees (Apis melifera) were introduced to North America to provide a source of honey for European colonists. This makes honeybees a(n):

A
Native species
B
Exotic species
C
Threatened species
D
Invasive species
Question 7 Explanation: 
The correct answer is (B). Honeybees are exotic species (also known as alien species) because they live outside their historical range. Native species have lived in their historical range for thousands of years. Threatened species have a high risk of extinction in the future. Invasive species and invasive alien species are the same thing. They are non-native species capable of causing significant harm to the environment, the economy, or society.
Question 8

What causes biodiversity loss worldwide?

A
Pollution
B
Habitat loss
C
Climate change
D
All of the above
Question 8 Explanation: 
The correct answer is (D). All of the following have contributed to the worldwide loss of biodiversity: overharvesting, pollution, habitat loss, and climate change.
Question 9

Which greenhouse gas is correctly paired with its source?

A
Carbon dioxide; burning coal and oil
B
Chlorofluorocarbons; fire extinguishers
C
Nitrous oxide; cellular respiration
D
Methane; plastic manufacturing
Question 9 Explanation: 
The correct answer is (A). Carbon dioxide comes from burning coal and oil, as well as deforestation and cellular respiration. Chlorofluorocarbons are from air conditioners, refrigerators, foam products and insulation. Nitrous oxide is from burning fossil fuels, fertilizers, livestock wastes, and plastic manufacturing. Methane is from rice cultivation, cattle/sheep raising, landfills, natural gas leaks, and coal production. Sulfur hexafluoride is from the electrical industry as a replacement for PCBs.
Question 10

Over the past 100 years, global temperatures have:

A
increased, with virtually of all the increase occurring in the last 40 years
B
fluctuated within a narrow range, with no net increase
C
increased, with most of the increase occurring from 1940 to 1980
D
decreased steadily, before a sharp increase in the last 20 years
Question 10 Explanation: 
The correct answer is (A). Over the past 100 years, average global temperatures have risen by a little more than 1° C. The trend of increasing temperatures began in the late '70s. 17 of the 18 warmest years in the 136-year record happened after 2001.
Question 11

Which of the following agreements was designed to protect the ozone layer by phasing out the global production of numerous substances which cause ozone depletion?

A
Clean Air Act
B
Kyoto Protocol
C
Paris Agreement
D
Montreal Protocol
Question 11 Explanation: 
The correct answer is (D). The Montreal Protocol is an international agreement designed to protect the stratospheric ozone layer.

The Clean Air Act was passed in 1963 and was designed to control air pollution on a national level.

The Kyoto Protocol was a plan created by the United Nations that tries to reduce the effects of climate change.

The Paris Agreement set out a global action plan to put the world on track to avoid dangerous climate changes.

The Air Pollution Control Act addressed air pollution as a national problem and announced that research and additional steps to improve the situation needed to be taken.
Question 12

Which of the following was the 1992 UN treaty to reduce greenhouse gases through binding emission reduction targets on developed countries?

A
Clean Air Act
B
Kyoto Protocol
C
Paris Agreement
D
Montreal Protocol
Question 12 Explanation: 
The correct answer is (B). The Kyoto Protocol was a United Nations plan to fight global warming by reducing greenhouse gas concentrations in the atmosphere. The agreement put the obligation to reduce emissions on developed countries, mandating that 37 industrialized nations plus the European Community cut their greenhouse gas emissions. More than 100 developing countries, including China and India, were not required to reduce emissions. The United States never ratified the agreement and Canada withdrew from it in 2011.
Question 13

Which of the following was the 2016 UN agreement on greenhouse gas emissions, allowing each country to determine, plan, and report on its contribution with no enforcement mechanism?

A
Clean Air Act
B
Kyoto Protocol
C
Paris Agreement
D
Montreal Protocol
Question 13 Explanation: 
The correct answer is (C). The Paris Agreement states that the threat of climate change is "urgent and potentially irreversible" and can only be addressed through "the widest possible cooperation by all countries" and "deep reductions in global emissions." The goal of the agreement is to hold the increase in global average temperature to well below 2 degrees C above pre-industrial levels. Each country submitted a voluntary pledge to reduce emissions beginning in 2020. In June 2017, President Trump announced his intention to withdraw the US from the agreement.
Question 14

Which of the following is NOT a naturally occurring greenhouse gas?

A
Methane
B
Nitrous oxide
C
Chlorofluorocarbons
D
Carbon dioxide
Question 14 Explanation: 
The correct answer is (C). There are no known naturally occurring sources of chlorofluorocarbons (CFCs).
Question 15

What was the primary purpose of reintroducing wolves to Yellowstone in the 1990s?

A
Restore the natural ecosystem of the park by keeping herbivores in check and allowing plant life to recover
B
Introduce a competitor for the coyotes because they were dominating the ecosystem
C
Attract more tourists to view the wolves
D
Protect livestock that inhabits the outer edges of the park
Question 15 Explanation: 
The correct answer is (A). Wolves are keystone species that help to establish a balance between producers and primary consumers in the same ecosystem they are found living in. Without the wolves, elk and deer were decimating the local plant life, reducing the primary productivity of the ecosystem.
Question 16

Which of the following is NOT a possible impact of global warming?

A
Disease vectors will find increased ranges as the climate warms near the poles
B
Flood plains used to grow crops will be under water due to sea level rise
C
Areas now suitable for agriculture will become even more productive
D
People will need to migrate to new regions as local water supplies are disrupted
Question 16 Explanation: 
The correct answer is (C). It is more likely that growing regions will shift towards the poles and current agriculture sites will experience drought-like conditions.
Question 17

Why are HCFCs NOT a permanent solution to protect the stratospheric ozone layer?

A
They are just as useful as refrigerants but not in sprays
B
HCFCs do not harm the ozone layer but do contribute to global warming
C
HCFCs do not contribute to global warming but do harm the ozone layer
D
HCFCs still harm the ozone layer, just not as much
Question 17 Explanation: 
The correct answer is (D). HCFCs are less harmful to the ozone layer but still cause harm. Other solutions do not harm the ozone layer, but some lead to more global warming. All countries are required to phase out HCFCs by 2030.
Question 18

Ozone depletion occurs naturally over Antarctica during which season?

A
Spring
B
Summer
C
Fall
D
Winter
Question 18 Explanation: 
The correct answer is (A). During the Antarctic spring, rising temperatures in the atmosphere allow molecules to react with solar radiation and release Cl atoms, which then break down ozone.
Question 19

Human activities increase the rate of evaporation of water into the atmosphere, but anthropogenic water vapor does not contribute to climate change because:

A
Water vapor does not absorb and re-radiate infrared
B
Water vapor is not a greenhouse gas
C
Heat absorbed by water vapor is only directed back out to space
D
Water vapor has a short residence time in the atmosphere
Question 19 Explanation: 
The correct answer is (D). Water vapor does not remain in the atmosphere for long before falling back to the Earth as precipitation.
Question 20

Increased carbon dioxide in the atmosphere will impact marine organisms in what way?

A
pH will decrease outside the range of tolerance for most marine organisms
B
Water temperature will decrease, causing marine organisms to migrate towards the equator
C
There would be decreased availability of bicarbonate ions, making it harder for corals and shellfish to grow their shells
D
There would be increased availability of bicarbonate ions, making it harder for corals and shellfish to grow their shells
Question 20 Explanation: 
The correct answer is (C). Increased carbon dioxide will lead to bicarbonate ions getting bound up by carbonic acid. Corals and shellfish require bicarbonate ions to build their shells and skeletons. pH will increase, not decrease, and ocean temperatures will likewise increase.
Question 21

Which would likely lead to an increase in the acidity of the oceans?

A
Increased burning of fossil fuels for power generation.
B
Decrease in deforestation for agriculture.
C
Increase in primary productivity of terrestrial ecosystems.
D
Increase in the use of public transportation.
Question 21 Explanation: 
The correct answer is (A). Burning of fossil fuels adds CO2 to the atmosphere which then is absorbed by the oceans. CO2 in the ocean forms carbonic acid which is harmful to marine invertebrates preventing them from growing shells. The other three options would reduce CO2 emissions.
Question 22

This greenhouse gas can be found in the lower troposphere due to photochemical reactions.

A
H2O
B
CO2
C
O3
D
CH4
Question 22 Explanation: 
The correct answer is (C). Ozone in the troposphere is due to reactions between NOx, VOCs, and sunlight. Ozone is a secondary pollutant while the other three options are primary pollutants. Secondary pollutants form as a result of chemical reactions in the atmosphere.
Question 23

Which of the following is a contributor to both global warming and depletion of the ozone layer?

A
Increased CO2 in the atmosphere from burning fossil fuels.
B
Release of CFCs into the atmosphere.
C
Increased levels of UV light reaching the surface of the Earth.
D
Increased methane in the atmosphere from pre-industrial levels.
Question 23 Explanation: 
The correct answer is (B). CFCs both lead to the depletion of stratospheric ozone and act as a potent greenhouse gas. Carbon dioxide and methane do not play a part in the reduction of the ozone layer.
Question 24

Which of the following would likely have the greatest positive impact on ecosystems globally?

A
Stabilization or reduction of the size of the human population.
B
Increased agricultural production in marginal farming communities.
C
Increased life expectancy in developing nations.
D
Discovery of more fossil fuel reserves in arctic regions.
Question 24 Explanation: 
The correct answer is (A). If the human population stops growing improvements in technology and agriculture would allow us to gradually reverse some of the negative impacts we have caused as a species.
Question 25

The data shows global sea levels have risen in the last 100 years.  Which of the following factors best explains why this has happened?

A
Increased precipitation.
B
Melting permafrost.
C
Increased evapotranspiration.
D
Thermal Expansion.
Question 25 Explanation: 
The correct answer is (D). Roughly 40% of sea level rise can be attributed to the fact that warm water is lower density and takes up more “space” than cooler water. Precipitation brings more water onto the land from the oceans.
Question 26

If average global temperatures continue to rise as predicted, which of the following is most likely?

A
The geographical range of plants will move towards the equator.
B
There will be a decline in pests and disease-carrying vectors near the poles.
C
Specialist species will be at greater risk of extinction
D
Specialist species will be at a lesser risk of extinction.
Question 26 Explanation: 
The correct answer is (C). Specialist species are less likely to adapt to a changing environment brought on by global climate change. The geographic range of plants will move towards the poles, not the equator. Insects and other pests will spread towards the poles as the climate becomes more moderate.
Question 27

Which of the following best describes the process involved in the greenhouse effect?

A
Ozone absorbs UV light from the sun warming the stratosphere.
B
Absorption of infrared radiation by atmospheric gases in the troposphere.
C
Diffusion of visible light by aerosols.
D
Breakdown of oxygen in the thermosphere.
Question 27 Explanation: 
The correct answer is (B). Greenhouse gases are exceptional at absorbing infrared radiation and re-radiate the heat back towards the Earth and out into space. More greenhouse gases mean more trapped heat. The other three do not have an impact on the Earth’s climate.
Question 28

An area of temperate forest will be partially cleared to build a new subdivision.  The developers will leave some sections of forest intact to preserve as much of the native ecosystem as possible.  Which is the likely impact of the removal of much of the forest ecosystem?

A
An increase in large predators making use of more vulnerable prey populations.
B
A decrease in the rate of soil erosion in the area.
C
An increase in the rate of primary productivity in the area.
D
A reduction of specialist species due to smaller habitats.
Question 28 Explanation: 
The correct answer is (D). Habitat fragmentation puts more stress on animals in higher trophic levels and on specialist species who have narrow niche requirements.
Question 29

The most likely cause of species extinction in the future is likely:

A
Habitat loss
B
Hunting and poaching
C
Weaker environmental legislation
D
Aggressive collection for museums and zoos
Question 29 Explanation: 
The correct answer is (A). While all four options are harmful to species diversity, habitat destruction is the most impactful. The destruction of natural habitats for agriculture, urban expansion, natural resource acquisition, and logging continues as the human population grows.
Question 30

Scientists theorize that increased carbon dioxide in the oceans could be harmful to certain fish species.  They suspect immature fish are more at risk due to the increased carbon dioxide.  Which of the following could be a good, testable hypothesis to test this theory?

A
Increased carbon dioxide in the water will lead to denser skeletons in mature fish.
B
Decreasing the pH of the water will lower fish larvae survival rates.
C
Decreasing the temperature of the water will lead to lower fish larvae survival rates.
D
There will be a greater number of fish larvae that survive than mature fish that survive.
Question 30 Explanation: 
The correct answer is (B). Increased levels of carbon dioxide in the ocean water lowers pH by creating carbonic acid. If fish larvae survive at lower rates at a lower pH this would support the original hypothesis.
Once you are finished, click the button below. Any items you have not completed will be marked incorrect. Get Results
There are 30 questions to complete.
List
Return
Shaded items are complete.
12345
678910
1112131415
1617181920
2122232425
2627282930
End
Return